Linear Integration of a Vector Field over a Parametric Path

Click For Summary
The discussion centers on the challenges of integrating a vector field over a parametric path, with the proposed method involving the dot product of the vector field F and the derivative of the path. Participants note that this approach leads to a complicated integral, ultimately arriving at the answer of 3/e. There is a suggestion that if the vector field F were the gradient of a scalar function, the problem would be simpler, prompting a search for such a function. Concerns are raised about the timing of the problem in relation to the course material, specifically the introduction of gradients. Overall, the discussion highlights the complexity of the integration process and the potential for alternative solutions.
sriracha
Messages
28
Reaction score
0
Problem statement attached. The correct way to do this seems to plug in your given x, y, z into F then integrate the dot product of F and <x',y',z'> dp from 0 to 1, however, this results in way too messy of an integral. Answer is 3/e.

<e^-(sin(pi*p/2))-((1-e^p)/(1-e))e^-(ln(1+p)/ln(2)),e^-((1-e^p)/(1-e))-(ln(1+p)/ln(2))e^-(sin(pi*p/2)),e^-(ln(1+p)/ln(2))-(sin(pi*p/2))e^-((1-e^p)/(1-e))>.<2^(x-1)ln(2),2/(pi*sqrt(1-y^2),(e-1)/((e-1)y+1)>
 

Attachments

  • III-19.jpeg
    III-19.jpeg
    33 KB · Views: 525
Physics news on Phys.org
sriracha said:
Problem statement attached. The correct way to do this seems to plug in your given x, y, z into F then integrate the dot product of F and <x',y',z'> dp from 0 to 1, however, this results in way too messy of an integral. Answer is 3/e.

<e^-(sin(pi*p/2))-((1-e^p)/(1-e))e^-(ln(1+p)/ln(2)),e^-((1-e^p)/(1-e))-(ln(1+p)/ln(2))e^-(sin(pi*p/2)),e^-(ln(1+p)/ln(2))-(sin(pi*p/2))e^-((1-e^p)/(1-e))>.<2^(x-1)ln(2),2/(pi*sqrt(1-y^2),(e-1)/((e-1)y+1)>

It would be awfully nice if the vector field F were a gradient of some scalar function, wouldn't it? Can you guess one that works?
 
Okay so I figured this out, but I had to read into the next chapter to do so. This is from Div, Grad, Curl, which really should be called Div, Curl, Grad. Why would Schey ask this problem before you get to gradient and how would he expect you to find it otherwise? Can anyone think of another possible path to solving this? I mean it's certainly possible that someone would realize the F.t ds = \psi, but I imagine even for a quite brilliant person that would take some time.
 
Question: A clock's minute hand has length 4 and its hour hand has length 3. What is the distance between the tips at the moment when it is increasing most rapidly?(Putnam Exam Question) Answer: Making assumption that both the hands moves at constant angular velocities, the answer is ## \sqrt{7} .## But don't you think this assumption is somewhat doubtful and wrong?

Similar threads

Replies
4
Views
2K
  • · Replies 14 ·
Replies
14
Views
2K
  • · Replies 11 ·
Replies
11
Views
1K
  • · Replies 5 ·
Replies
5
Views
1K
  • · Replies 18 ·
Replies
18
Views
3K
  • · Replies 8 ·
Replies
8
Views
2K
  • · Replies 5 ·
Replies
5
Views
1K
  • · Replies 3 ·
Replies
3
Views
1K
Replies
5
Views
2K
Replies
3
Views
2K